Άθροισμα γωνιών μικρότερο από 90

Συντονιστές: vittasko, silouan, rek2

Άβαταρ μέλους
Ορέστης Λιγνός
Δημοσιεύσεις: 1835
Εγγραφή: Κυρ Μάιος 08, 2016 7:19 pm
Τοποθεσία: Χαλάνδρι Αττικής
Επικοινωνία:

Άθροισμα γωνιών μικρότερο από 90

#1

Μη αναγνωσμένη δημοσίευση από Ορέστης Λιγνός » Σάβ Μάιος 06, 2017 8:48 pm

Θεωρούμε οξυγώνιο τρίγωνο ABC, με \widehat{C} \geqslant \widehat{B}+30^0.

Έστω O το περίκεντρο του \triangle ABC, και AP το ύψος.

Να δείξετε ότι \widehat{A}+\widehat{COP} <90^0.
ORESTIS-GWNIA.png
ORESTIS-GWNIA.png (9.18 KiB) Προβλήθηκε 1101 φορές


Κερδίζουμε ό,τι τολμούμε!

Λέξεις Κλειδιά:
Άβαταρ μέλους
ΣΤΑΘΗΣ ΚΟΥΤΡΑΣ
Επιμελητής
Δημοσιεύσεις: 4658
Εγγραφή: Κυρ Μαρ 13, 2011 9:11 pm
Τοποθεσία: Βρυξέλλες

Re: Άθροισμα γωνιών μικρότερο από 90

#2

Μη αναγνωσμένη δημοσίευση από ΣΤΑΘΗΣ ΚΟΥΤΡΑΣ » Τετ Μάιος 10, 2017 10:40 pm

Ορέστης Λιγνός έγραψε:Θεωρούμε οξυγώνιο τρίγωνο ABC, με \widehat{C} \geqslant \widehat{B}+30^0.Έστω O το περίκεντρο του \triangle ABC, και AP το ύψος.Να δείξετε ότι \widehat{A}+\widehat{COP} <90^0.
Έστω ότι η μεσοκάθετη της BC (διερχόμενη προφανώς από το περίκεντρο O ) τέμνει την AB στο σημείο T την BC στο M και ας είναι

E\equiv CE\cap \left( O \right),E\ne C και N\equiv OT\cap EA . Τότε το τετράπλευρο AEBC είναι προφανώς ισοσκελές τραπέζιο οπότε NA=MP και

\angle ECA = \angle C - \angle ECB\mathop  = \limits^{TC = TB} \angle C - \angle B \geqslant {30^0}\mathop  \Rightarrow \limits^{\varepsilon \pi \iota \kappa \varepsilon \nu \tau \rho \eta  - \varepsilon \gamma \gamma \varepsilon \gamma \rho \alpha \mu \mu \varepsilon \nu \eta } \angle EOA \geqslant {60^0} \Rightarrow EA \geqslant R\mathop  \Rightarrow \limits^{\left( 1 \right)} \boxed{MP \geqslant \frac{{OC}}{2}}:\left( 2 \right).
[attachment=0]Αθροισμα μεγαλύτερο του 90.png[/attachment]
Είναι P{C^2} = {\left( {MC - MP} \right)^2} = M{C^2} + M{P^2} - 2MC \cdot MP = O{C^2} - O{M^2} + O{P^2} - O{M^2} - 2MC \cdot MP \Rightarrow

O{P^2} - P{C^2} = 2O{M^2} - O{C^2} + 2MC \cdot MP \geqslant 2O{M^2} - O{C^2} + MC \cdot OC =

O{M^2} - M{C^2} + MC \cdot OC\mathop  > \limits^{OC > MC} O{M^2} > 0 \Rightarrow O{P^2} - P{C^2} > 0 \Rightarrow OP > PC

\mathop  \Rightarrow \limits^{\vartriangle OPC} \angle OCP > \angle POC \Rightarrow {90^0} - \angle A > \angle POC \Rightarrow \boxed{\angle A + \angle POC < {{90}^0}}.
και το ζητούμενο έχει αποδειχθεί.

Στάθης
Συνημμένα
Αθροισμα μεγαλύτερο του 90.png
Αθροισμα μεγαλύτερο του 90.png (26.93 KiB) Προβλήθηκε 1020 φορές


Τι περιμένετε λοιπόν ναρθεί , ποιόν καρτεράτε να σας σώσει.
Εσείς οι ίδιοι με τα χέρια σας , με το μυαλό σας με την πράξη αν δεν αλλάξετε τη μοίρα σας ποτέ της δεν θα αλλάξει
Άβαταρ μέλους
Ορέστης Λιγνός
Δημοσιεύσεις: 1835
Εγγραφή: Κυρ Μάιος 08, 2016 7:19 pm
Τοποθεσία: Χαλάνδρι Αττικής
Επικοινωνία:

Re: Άθροισμα γωνιών μικρότερο από 90

#3

Μη αναγνωσμένη δημοσίευση από Ορέστης Λιγνός » Τετ Μάιος 10, 2017 11:00 pm

ΣΤΑΘΗΣ ΚΟΥΤΡΑΣ έγραψε:
Ορέστης Λιγνός έγραψε:Θεωρούμε οξυγώνιο τρίγωνο ABC, με \widehat{C} \geqslant \widehat{B}+30^0.Έστω O το περίκεντρο του \triangle ABC, και AP το ύψος.Να δείξετε ότι \widehat{A}+\widehat{COP} <90^0.
Έστω ότι η μεσοκάθετη της BC (διερχόμενη προφανώς από το περίκεντρο O ) τέμνει την AB στο σημείο T την BC στο M και ας είναι

E\equiv CE\cap \left( O \right),E\ne C και N\equiv OT\cap EA . Τότε το τετράπλευρο AEBC είναι προφανώς ισοσκελές τραπέζιο οπότε NA=MP και

\angle ECA = \angle C - \angle ECB\mathop  = \limits^{TC = TB} \angle C - \angle B \geqslant {30^0}\mathop  \Rightarrow \limits^{\varepsilon \pi \iota \kappa \varepsilon \nu \tau \rho \eta  - \varepsilon \gamma \gamma \varepsilon \gamma \rho \alpha \mu \mu \varepsilon \nu \eta } \angle EOA \geqslant {60^0} \Rightarrow EA \geqslant R\mathop  \Rightarrow \limits^{\left( 1 \right)} \boxed{MP \geqslant \frac{{OC}}{2}}:\left( 2 \right).
[attachment=0]Αθροισμα μεγαλύτερο του 90.png[/attachment]
Είναι P{C^2} = {\left( {MC - MP} \right)^2} = M{C^2} + M{P^2} - 2MC \cdot MP = O{C^2} - O{M^2} + O{P^2} - O{M^2} - 2MC \cdot MP \Rightarrow

O{P^2} - P{C^2} = 2O{M^2} - O{C^2} + 2MC \cdot MP \geqslant 2O{M^2} - O{C^2} + MC \cdot OC =

O{M^2} - M{C^2} + MC \cdot OC\mathop  > \limits^{OC > MC} O{M^2} > 0 \Rightarrow O{P^2} - P{C^2} > 0 \Rightarrow OP > PC

\mathop  \Rightarrow \limits^{\vartriangle OPC} \angle OCP > \angle POC \Rightarrow {90^0} - \angle A > \angle POC \Rightarrow \boxed{\angle A + \angle POC < {{90}^0}}.
και το ζητούμενο έχει αποδειχθεί.

Στάθης
:coolspeak:


Κερδίζουμε ό,τι τολμούμε!
Άβαταρ μέλους
Doloros
Επιμελητής
Δημοσιεύσεις: 9850
Εγγραφή: Τρί Αύγ 07, 2012 4:09 am
Τοποθεσία: Ιεράπετρα Κρήτης

Re: Άθροισμα γωνιών μικρότερο από 90

#4

Μη αναγνωσμένη δημοσίευση από Doloros » Πέμ Μάιος 11, 2017 10:53 am

Ορέστης Λιγνός έγραψε:Θεωρούμε οξυγώνιο τρίγωνο ABC, με \widehat{C} \geqslant \widehat{B}+30^0.

Έστω O το περίκεντρο του \triangle ABC, και AP το ύψος.

Να δείξετε ότι \widehat{A}+\widehat{COP} <90^0.

ORESTIS-GWNIA.png
Πρώτα –πρώτα ας δούμε πως πρέπει να στηθεί ένα τρίγωνο ABC, με τις προδιαγραφές της

υπόθεσης .

Λήμμα_Αθροισμα γωνιών μικρότερο απο 90.png
Λήμμα_Αθροισμα γωνιών μικρότερο απο 90.png (26.2 KiB) Προβλήθηκε 937 φορές
Έστω κύκλος (O,R) μια διάμετρος TS και μια χορδή BC//TS . Για να έχουμε

εγγεγραμμένο οξυγώνιο τρίγωνο \vartriangle ABC, θα πρέπει το σημείο A σε πρώτη φάση να

ανήκει στο μεγάλο τόξο \tau o\xi BC. Όμως πρέπει επί πλέον \widehat C \geqslant \widehat B + 30^\circ .

Προς τούτο γράφω τον κύκλο (B,R) που τέμνει το μεγάλο τόξο \widehat C = \widehat B + 30^\circ\tau o\xi BC

στο σημείο D άρα \widehat {DCB} = 30^\circ. Φέρνω τη μεσοκάθετο στη χορδή CD που τέμνει το

μεγάλο τόξο \tau o\xi BC στο σημείο M και θα είναι \widehat {SOM} = 60^\circ . Ας είναι N το μέσο

του BC. αν P η προβολή του M( ή όποιου άλλου σημείου A του τόξου \tau o\xi SM)

στο BC θα είναι προφανώς NP > PC\,\,\kappa \alpha \iota \,\,\widehat {ACB} > 30^\circ  + \widehat B , ενώ αν A \equiv M θα είναι

\widehat C = \widehat B + 30^\circ.

Μετά απ’ αυτά θεωρώ C' το συμμετρικό του C ως προς τη TS και το A θα ανήκει

στο τόξο \tau o\xi MC' αλλά χωρίς το C'.
Αθροισμα γωνιών μικρότερο απο 90.png
Αθροισμα γωνιών μικρότερο απο 90.png (44.76 KiB) Προβλήθηκε 937 φορές
Θα είναι δε OP > NP > PC \Rightarrow OP > PC \Leftrightarrow \widehat {{x_1}} > \widehat \theta  \Rightarrow \widehat {{x_2}} > \widehat \theta και έτσι :

\widehat {NOC} + \widehat \theta  < \widehat {NOC} + \widehat {{x_2}} \Rightarrow \boxed{\widehat A + \widehat \theta  < 90^\circ }


Άβαταρ μέλους
silouan
Επιμελητής
Δημοσιεύσεις: 1398
Εγγραφή: Τρί Ιαν 27, 2009 10:52 pm

Re: Άθροισμα γωνιών μικρότερο από 90

#5

Μη αναγνωσμένη δημοσίευση από silouan » Πέμ Μάιος 11, 2017 9:30 pm

Είναι από την ΙΜΟ 2001. https://artofproblemsolving.com/communi ... 462p119192


Σιλουανός Μπραζιτίκος
Άβαταρ μέλους
Doloros
Επιμελητής
Δημοσιεύσεις: 9850
Εγγραφή: Τρί Αύγ 07, 2012 4:09 am
Τοποθεσία: Ιεράπετρα Κρήτης

Re: Άθροισμα γωνιών μικρότερο από 90

#6

Μη αναγνωσμένη δημοσίευση από Doloros » Πέμ Μάιος 11, 2017 10:29 pm

silouan έγραψε:Είναι από την ΙΜΟ 2001. https://artofproblemsolving.com/communi ... 462p119192
Να ευχαριστήσω το Κ. Μπραζιτίκο για την ενημέρωση σχετικά με την άσκηση.

Θα έλεγα ότι, όταν οι ασκήσεις που δίδονται είναι αυτούσιες από διαγωνισμούς, πρέπει από τους εισηγητές να δίδονται στο τέλος και οι πηγές .


Απάντηση

Επιστροφή σε “Γεωμετρία - Επίπεδο Αρχιμήδη (Seniors)”

Μέλη σε σύνδεση

Μέλη σε αυτήν τη Δ. Συζήτηση: Δεν υπάρχουν εγγεγραμμένα μέλη και 1 επισκέπτης